Binomial Sums / Mellin Asymptotics with Explicit Error Bounds
Benjamin Hackl • June 20, 2024

Binomial Sums and Mellin Asymptotics
with Explicit Error Bounds

Case Study & Interactive Software Demo

Joint work with Stephan Wagner (TU Graz)

AofA2024 @ University of Bath, UK
This work is licensed under a Creative Commons Attribution-NonCommercial-ShareAlike 4.0 International License.

Act I — Motivating Example

  • Max-peak-unique Dyck paths: Dyck paths with unique peak of max. height
Problem (Bóna, DeJonge) Is $\frac{\# \text{max-peak-unique Dyck paths of length } 2n}{C_n}$ decreasing for $n \geq 3$?

Enumerating max-peak-unique Dyck paths

  • Cut in half: split max-peak-unique Dyck path at peak, obtain 2 "culminating paths"
  • OGF of culminating paths ending at height $h$: $\frac{1}{U_h(1 / (2x))}$ (Thanks, Clemens!)
Proposition If $a_n$ ... # max-peak-unique Dyck paths of length $2n$, then \[ A(z) = \sum_{n\geq 0} a_n x^{2n} = \sum_{h\geq 0} \frac{1}{U_h(1/(2x))^2} \]

From Coefficient Extraction ...

  • Ingredients:
    • $U_h(\cosh w) = \sinh((h+1) w) / \sinh w$
    • $x = \sqrt{t} / (1 + t) \iff 1/(2x) = \cosh(\frac{1}{2} \log t)$

\[ \scriptsize\Rightarrow A(x) = \sum\limits_{h\geq 0} \frac{t^h (1 - t)^2}{(1 - t^{h+1})^2} \]

Proposition \[ a_n = \sum_{k=1}^{n+1} \frac{4 k \sigma(k) (2k^2 - 3n - 2) (2n-1)!}{(n+1-k)!\, (n+1+k)!} \]

... to an Inequality ...

  • Bóna—DeJonge-question $\iff a_n / C_n \gt a_{n+1} / C_{n+1}$ for $n\geq 3$
  • After some further manipulations, this is equivalent to...
Proposition $a_n / C_n$ is decreasing for $n \geq 3$ if and only if \[ F(n) := \sum_{k=1}^n k \sigma(k) (k^2 - 3n + 2)(2k^2 - n) \binom{2n}{n - k} \lt 0 \] for $n\geq 5$.

... followed by Asymptotics!

  • Split binomial sum into "central" and "tails" regions, show contribution
    of tails is negligible
  • Approximate $\binom{2n}{n-k}$ in "central" region, complete tails
  • Mellin for integral representation; shift line of integration + collect residues
Theorem (H.—Wagner, 2024) For $n \to \infty$, we have \[ F(n) = \binom{2n}{n} \Big(- \frac{n^2}{8} + \frac{n}{24} + o(n)\Big) \]
  • Not good enough: no information about error—can only say sequence is eventually decreasing...

Act IIdependent_bterms in SageMath

  • $B$-Terms: $O$-terms with explicit constant + validity point, e.g., $B_{n\geq 100}(42 n^3)$
  • Package extends SageMath; assume $n \to \infty$:
    • Monomially bounded variable $n^{\alpha} \leq k \leq n^{\beta}$
    • Taylor expansions with (and at) $B$-terms
  • Installation:
    sage -pip install dependent_bterms
  • Try it: behackl.dev/asy/dbt or static version

Creating an (enhanced) AsymptoticRing


			import dependent_bterms as dbt

			AR, n, k = dbt.AsymptoticRingWithDependentVariable(
			    'n^QQ',  # growth group, governs summand growth
			    'k', 0, 4/7,  # n^0 <= k <= n^(4/7)
			    bterm_round_to=3,  # number of decimal places kept when rounding
			    default_prec=5  # default precision for automatic expansions
			)
			
  • Summands are ordered w.r.t. largest potential growth:

BTerm Arithmetic with a dependent variable

\[ \scriptsize \frac{1 - 2k + 3k^2 - 4k^3}{n^5} + B_{n\geq 10}(3 k^2 / n^3) =\, ? \]

  • $k^3 n^{-5} = O(n^{-23/7}) = O(n^{-3})$ → absorbable!
  • Crude estimate: $\scriptsize |1 - 2k + 3k^2 - 4k^3| \leq (1 + 2 + 3 + 4)k^3 = 10k^3$
  • Use given bounds $k \leq n^{4/7}$ and $n\geq 10$:
    $\frac{10 k^3}{n^5} \leq \frac{10 k^2}{n^{31/7} } \leq \frac{10 k^2}{10^{10/7} n^3}$

Simplifications and Taylor Expansions

Act III — Towards an Explicit Error

  • Fix $N = 10000$, consider growth of $F(n)$ for $n \geq N$
  • Check inequality $F(n) \lt 0$ for $5 \leq n \lt N$

Tails: Identify range of negligible contributions

  • Normalize by central binomial coefficient, want to work with $\binom{2n}{n-k} / \binom{2n}{n}$ instead
  • Split sum over $1 \leq k\leq n$ into different regions:
    • $k \gt n/2$: ratio of binomial coefficients is exponentially small: $\scriptsize B_{n\geq N}(52/25\, e^{-n/4} n^7 \log\log n )$
    • $n^{\alpha} \leq k \leq n/2$ with $\alpha = 7/10$, sum bound: $\scriptsize 208/25 \log\log n \sum k^6 e^{-k^2 / n} $, then use integral estimate: $\scriptsize B_{n\geq N}(25073/5000\, e^{-n^{2/5} } n^{9/2} \log\log n)$

Dealing with Summands for $1\leq k \lt n^{\alpha}$

  • Determine expansion of \[ \binom{2n}{n-k} / \binom{2n}{n} = e^{-k^2/n} \Big( 1 + \frac{k^2}{n^2} - \frac{k^4 + k^2}{6 n^3} + ...\Big) \]
  • "Error part" contribution: $\scriptsize B_{n\geq N} (14671.8899\, \sqrt{n} \log\log n)$
  • "Exact part": tail completion ↝ (sub)exponentially decaying errors \[ B_{n\geq N}\Big(2.6\, e^{-n^{2/5} } n^{19/4} \log\log n + 0.002\, e^{-n^{1/2} } n^{11/2} \Big) \]

Extract Asymptotics!

  • Compute Mellin transform; converse mapping → shift line of integration, collect residues...
  • Main term (with heavy cancellations): $-n^2/8 + n/24$
  • Estimate shifted integrals:
    • $\Gamma$ causes fast decay, crude estimate for $\scriptsize|\operatorname{Im}(t)| \gt 100$
    • Numerical evaluation w/ interval arithmetic for $\scriptsize|\operatorname{Im}(t)| \leq 100$
    Error: $\scriptsize B_{n\geq N}(4065.31\, n^{3/4})$

Quantified Binomial Sum Asymptotics

After collecting and combining all error terms...

Theorem (H.—Wagner, 2024) \[ \scriptsize F(n) = \binom{2n}{n} \Big( -\frac{n^2}{8} + \frac{n}{24} + B_{n\geq 10000}\big(7751.1106\,n^{3/4}\big) \Big), \]

and $F(n) \lt 0$ for all $n\geq 5$.

Bonus: For any length $n$ there are at least as many 132-avoiding permutations with a unique longest increasing subsequence as those without.

Thank you!